返回列表 发帖

求教Prep07一道CR

不明白为什么C是正确选项。。。




13.     Which of the following most logically completes theargument?




United States manufacturers currently produce most of theworld's solar-power generators--most of which are exported to Europe.  However, European manufacturers are emergingand probably will ultimately capture much of the European market.  The United States government is funding initiativesintended to encourage use of solar power within the United States.  If these initiatives succeed in increasingthe demand for solar-power generators in the United States, United Statesmanufacturers will probably maintain significant production levels, since__________.




(A) some United States manufacturers have beensubstantially increasing their output over the last several years




(B) the efficiency of solar-power generators inconverting energy from the Sun into electric power is not improving as fast asit once did




(C) just as European manufacturers enjoy certaincompetitive advantages in Europe, so do United States manufacturers in theUnited States




(D) European governments are currently undertakinginitiatives to stimulate the use of solar power within Europe




(E) the current market for solar-power generators in theUnited States is very limited


收藏 分享

本来想用自己的话跟楼主解释,但是发现海外论坛上某人说得已经非常非常好了,copy过来一起学习
ok, one th ...
DXDWSY 发表于 2013-1-15 21:22



我可以排除E这个选项,但还是不是很懂那个gap,因为logical gap是我们在需要necessary assumption时,看要使结论成立,必要条件是什么。
e.g 经济发展使收入增加,结论:经济发展让人生活水平提高。这里的gap是收入增加必须使生活水平提高,是必要条件,取非后结论不能成立的。
本题中从manufacturer角度讲,要保持那个生产量,就是说从美国新增加的数量要大于或等于在欧洲市场失去的量。所以答案只需要给出这个必要的条件就行了。而不需要到competitive edge的程度。
比如美国政府这个政策可以带来非常巨大的新增demand,即使美国manufacturer没有获得competitive advantage,只要他们获得了其中一部分市场,然后这部分市场可以补回在欧洲的损失就行了。
我觉得这道题的解题方法不能是fill the gap,因为gap是必要条件的缺失
C选项提供的是一个充分条件,而不是必要条件。


E为什么不能选,首先,现在市场limited,最多暗示了US gov可以成功地增加demand,这个题目已经给出了肯定会成功。其次因为题目已经给出了if US gov stimulate demand,增加demand可以是从limited增加到很多,也可以是很多增加到更多,这些manufacturer需要的只是有增加,而不是增加的起点。

TOP

谢谢 谢谢 。明白了 特别是C和E之间,我也是感觉GMAC出题尽量不引入额外的因素 阅读也是这样。

TOP

本来想用自己的话跟楼主解释,但是发现海外论坛上某人说得已经非常非常好了,copy过来一起学习
ok, one thing should be clear at the outset: you're trying to fill in a gap in the logic. (in fact, there's a literal gap to fill in!)

here's a diagram type thingy:
euro companies emerge
AND
euro companies take over euro market
AND
u.s. market emerges
AND
----
THEREFORE
u.s. companies will replace euro market sales with u.s. market sales

we currently have facts asserting that the u.s. firms will lose their hold on the euro market. however, we have nothing asserting that the u.s. firms will gain a hold on the u.s. market, the second half of the conclusion.

therefore, we need an assertion that says, or implies, that the u.s. firms will gain a hold in the u.s. market (i.e., the euro firms won't take over the u.s. market as well).

(a) irrelevant:
- recent increases are unrelated to maintaining production in the future
- recent increases in production have nothing to do with the main logic gap (ensuring that u.s. firms will have a hold on the u.s. market)

(b) irrelevant:
- this has nothing to do with anything related to the argument

(c) correct
this statement strongly suggests that the u.s. firms will dominate the u.s. market for the same reasons that the euro firms will dominate the euro market.

(d) doesn't help
- we know that the u.s. firms will lose the euro market anyway, so it doesn't matter whether that market grows or not

(e) WEAKENS the argument
- if the u.s. market is very small, then it's likely that the production/revenue/etc of u.s. firms will decline rather than stay constant.

--

as far as you query 'what is the best way to arrive at the correct answer': there's clearly not an extremely simple way, since gmac would not bother writing these questions if there were. the best way to figure out the underlying logic is to make a diagram (like the one i made above, with the ands and therefores), which will force you to map out the logic.


如果楼主也和我一样犹豫C和E,那么看这里

I dint like C.. because... C was more vague compared to E..


here's something to keep in mind when you do these.

if you're torn between two answers on Strengthen/Weaken, you should try to PICK THE ANSWER THAT'S MORE DIRECTLY RELATED TO THE SPECIFIC ELEMENTS OF THE PASSAGE'S LINE OF REASONING.

in this case, here are the "specific elements" i'm talking about:
this argument is designed to show that
* american manufacturers will still maintain domestic market share,
* even with the encroachment of the european manufacturers on the market.

the problem with the choice you selected is that it has nothing to do with the latter of these two factors. in addition, as mentioned in my post above, if the american market is very small to start with, then this factor works against the argument (even if that small market grows so that it's ... not so small).

TOP

返回列表

站长推荐 关闭


美国top10 MBA VIP申请服务

自2003年开始提供 MBA 申请服务以来,保持着90% 以上的成功率,其中Top10 MBA服务成功率更是高达95%


查看